LSAT and Law School Admissions Forum

Get expert LSAT preparation and law school admissions advice from PowerScore Test Preparation.

User avatar
 Dave Killoran
PowerScore Staff
  • PowerScore Staff
  • Posts: 5852
  • Joined: Mar 25, 2011
|
#71217
This game is discussed in our Podcast: LSAT Podcast Episode 31: The September 2019 LSAT Logic Games Section


Complete Question Explanation
(The complete setup for this game can be found here: https://forum.powerscore.com/lsat/viewtopic.php?t=31306)

The correct answer choice is (C).

If G is in case 6, then the initial diagram appears as:

  • G H J M N P S 7



    ___ ..... ___ ..... ___ ..... ___ ..... ___ ..... _G_ ..... ___
     1 .....     2 .....    3 .....    4 .....    5 .....    6 .....      7
What are the consequences of G, the random, appearing in case 6? It's right next to case 7, which is limited to H or J, so examining case 7 makes some sense. In fact, for most people the best approach is to make some hypothetical solutions and see what occurs. Let's start with H in 7:

  • ___ ..... ___ ..... ___ ..... ___ ..... ___ ..... _G_ ..... _H_
     1 .....     2 .....    3 .....    4 .....    5 .....    6 .....      7
When G is in 7 and H is in 7, then to conform to the third rule M must be in case 1:

  • _M_ ..... ___ ..... ___ ..... ___ ..... ___ ..... _G_ ..... _H_
     1 .....     2 .....    3 .....    4 .....    5 .....    6 .....      7

But this is an issue since there is no way to conform to the second rule. Thus, it cannot be H in case 7 in this problem. Thus, J must be in case 7:

  • ___ ..... ___ ..... ___ ..... ___ ..... ___ ..... _G_ ..... _J_
     1 .....     2 .....    3 .....    4 .....    5 .....    6 .....      7
Immediately, check the answers to see if G is listed, since we now know that G must be next to J! Alas, there is no G, so let's continue on.

With J in 7, we can eliminate P and S from case 1 due to the last rule (since cases 1 and 7 are next to each other). This helpfully eliminates answer choices (D) and (E). Now, which artifacts remain to fill in case 1? G, J, P, and S, are eliminated, leaving only N, M, and H as options. Of course, we know that those three artifacts are linked in an N :longline: MH relationship, so, as discussed in the analysis of rules #2 and #3 in the setup, M and H can never be in case 1. This leaves only N available to fill case 1. Thus, with N in case 1 and J in case 7, N is next to J and answer choice (C) is correct.



Answer choice (A): As discussed in the analysis of rules #2 and #3 in the setup, H can never be in case 1 and so this answer choice is incorrect.

Answer choice (B): As discussed in the analysis of rules #2 and #3 in the setup, M can never be in case 1 and so this answer choice is incorrect.

Answer choice (C): This is the correct answer choice, as explained above.

Answer choice (D): This answer choice is eliminated by the interaction of the last rule and the fact that J must be in case 7 in this problem.

Answer choice (E): This answer choice is eliminated by the interaction of the last rule and the fact that J must be in case 7 in this problem.

Get the most out of your LSAT Prep Plus subscription.

Analyze and track your performance with our Testing and Analytics Package.